shariferguson
Thanks Received: 0
Forum Guests
 
Posts: 7
Joined: June 17th, 2010
 
 
 

Q5 - Doctor: While a few alternative

by shariferguson Wed Oct 13, 2010 2:05 pm

I guess this question is pretty straight forward given that is a number 5, but for some reason the logic behind why A is correct kind of bogles me.

So I have the conclusion is: Should always be allowed to prescribe
Why: - because it does not harm the patient and it might help.

How does A weaken it?

Thanks for your help in advance.
 
cyruswhittaker
Thanks Received: 107
Forum Guests
 
Posts: 246
Joined: August 11th, 2010
 
This post thanked 3 times.
 
trophy
Most Thanked
trophy
First Responder
 

Re: Q5 - Doctor: While a few alternative

by cyruswhittaker Wed Oct 13, 2010 11:06 pm

Even though this is an early question, it certainly can still be tricky if read too quickly.

Breakdown of the Argument:

The argument provides a premise (first sentence) indicating that, in regards to dangerous side effects, some alternative medicines have been proven safe. Just a detail note, but notice how the author is restricting the group to only "some" of such medicines. This is what the argument is pertaining to, and I've noticed some LSAT questions where they will provide an incorrect answer choice for a weaken choice by saying something like "author fails to recognize that not all..."

The author then claims that because some of these medicines have been proven safe, they should always be allowed. Notice that also added is "since their patients will not be harmed" in justification of the conclusion.

But the question is this: Are the side effects the only way that a patient could be harmed? For the argument to hold, then this would need to be the case. Hence, this is an unstated assumption.

Answer choice (A) directly attacks this assumption by providing evidence to show that it is questionable.
User avatar
 
LSAT-Chang
Thanks Received: 38
Atticus Finch
Atticus Finch
 
Posts: 479
Joined: June 03rd, 2011
 
 
trophy
Most Thankful
trophy
First Responder
 

Re: Q5 - While a few alternative medicines

by LSAT-Chang Fri Sep 23, 2011 2:11 pm

Hmm.. I'm sorry but can someone please explain to me why answer choice (A) weakens the argument? FOr some reason, I cannot follow the reasoning above. I can see that the doctor is concluding that herbal remedies for serious illnesses should ALWAYS be allowed. Why? Because the patients will not be harmed and might be helped. I don't understand what changes when we have many practitioners and patients neglecting more effective conventional medicines in favor of herbal remedies. Wouldn't that strengthen it? It seems like it would support the idea that herbs as remedies should always be allowed! I'm so confused..
 
dorbathedogslayer
Thanks Received: 2
Forum Guests
 
Posts: 4
Joined: September 23rd, 2011
 
 
 

Re: Q5 - While a few alternative medicines

by dorbathedogslayer Tue Sep 27, 2011 3:06 am

I'm a little confused about this as well.

I don't buy (A) simply because the fact that some practitioners don't use their freedom to prescribe alternative medicines does not mean that the licence to prescribe alternative medicines is a problem in itself. An analogous argument would imply that the presence of some poor doctors who sometimes misdiagnose patients should be an argument for why no doctor should be allowed to diagnose a patient.

(C) however, to me undermines the claim that "patients will not be harmed ... by the use of these products"

Is there something wrong with my logic?
 
vik
Thanks Received: 8
Jackie Chiles
Jackie Chiles
 
Posts: 42
Joined: March 29th, 2011
 
This post thanked 2 times.
 
 

Re: Q5 - While a few alternative medicines

by vik Tue Jan 24, 2012 8:53 pm

5 C does not weaken the argument because it could apply to conventional medicine too. It is not clear that C refers to herbal medicines. C would be a better weaken answer if it said dangerous reactions and herbal medicines.
 
evelina.chang
Thanks Received: 1
Forum Guests
 
Posts: 12
Joined: November 19th, 2011
 
 
 

Re: Q5 - Doctor: While a few alternative medicines

by evelina.chang Fri Feb 10, 2012 12:14 am

I suppose vik's explanation kind of clears it up for me, but I'm still confused why the correct answer is (A) - because it still seems to fit within the reasoning of the speaker to me.
User avatar
 
ManhattanPrepLSAT1
Thanks Received: 1909
Atticus Finch
Atticus Finch
 
Posts: 2851
Joined: October 07th, 2009
 
This post thanked 5 times.
 
 

Re: Q5 - Doctor: While a few alternative medicines

by ManhattanPrepLSAT1 Sun Feb 12, 2012 9:23 pm

I think I'll join the discussion on this one! Great discussion so far.

Conclusion: there's no reason not to allow doctors to prescribe these herbs as alternative medicines.

Evidence: these herbs are safe, and so while the medicinal effects are unproven, they won't cause harm.

Reasoning: suppose we were to question this conclusion. Can we think of a reason why the prescription of these herbs as alternative medicines should not be allowed? The first thing that might come to one's mind are side effects. But there's nothing to worry about there in this argument since we know the side effects are not harmful. So is there any other reason why it might be a bad idea to allow the prescription of these herbs as alternative medicines? Sure! What if people think that the alternative medicine is enough to treat their ailment? They'd be unlikely to seek further treatment as they would believe the ailment is being cured. For example, to treat a bacterial rash, you could take antibiotics (a conventional, effective treatment) or you could rub some herbs on it (an alternative remedy that will not cause any harm). If rubbing herbs makes someone less likely to take antibiotics, the conclusion that doctors should always be allowed to prescribe these herbs as alternative medicines would be undermined - stated nicely in answer choie (A).

Let's look at the incorrect answers:

(B) is consistent with the original argument. This makes it less likely that the herbal remedies will be effective, but the argument already concedes that the alternative remedies have little firm evidence of medicinal effect. This doesn't pose any harm to the patients, though they don't get the benefits they might expect.
(C) would apply to both alternative and conventional medicines.
(D) is irrelevant. That purveyors of herbal remedies are driven by the profit motive does not suggest a harm that could come about from being prescribed an alternative remedy.
(E) is consistent with the original argument that already conceded that there is little firm evidence of medicinal effect. This doesn't pose any harm to the patients. And really... who cares whether it's a placebo effect or an actual medicinal impact that creates the improvement for the patient?

Hope that helps! And again, nice work on this one.
 
wguwguwgu
Thanks Received: 5
Jackie Chiles
Jackie Chiles
 
Posts: 39
Joined: January 17th, 2012
 
 
 

Re: Q5 - Doctor: While a few alternative medicines

by wguwguwgu Sat Sep 01, 2012 8:36 pm

mattsherman Wrote:(C) would apply to both alternative and conventional medicines.


Hi Matt,
I picked A, it surely makes the most sense. However, I still don't understand why is C a bad answer just because it would apply to both alternative and traditional medicine. The conclusion is not saying that alternative medicine should be allowed, in contrary to traditional medicine. It only said that alternative medicine should be allowed.
I agree that C applies to both schools of medicine, but it would mean that neither alternative nor traditional medicine should be always allowed. And that does not conflict with the main conclusion, but rather includes and supports it, no?

thanks so much in advance
 
eapetrilli
Thanks Received: 5
Forum Guests
 
Posts: 11
Joined: August 06th, 2012
 
This post thanked 2 times.
 
 

Re: Q5 - Doctor: While a few alternative

by eapetrilli Sun Sep 09, 2012 5:55 pm

I made the mistake of choosing (C) under the timed pressure of the test. I think the answer to the last post might be found in the last part of the first sentence. The argument states as a given that certain herbal remedies have been proven safe to consume. Thus, these particular herbal remedies do not fall within the scope of answer choice (C). Therefore, we cannot weaken that direct claim about safety, but instead, must look elsewhere to weaken the expanded claim in the conclusion concerning their prescription.
 
timmydoeslsat
Thanks Received: 887
Atticus Finch
Atticus Finch
 
Posts: 1136
Joined: June 20th, 2011
 
This post thanked 1 time.
 
trophy
Most Thanked
trophy
First Responder
 

Re: Q5 - Doctor: While a few alternative

by timmydoeslsat Mon Sep 10, 2012 11:38 pm

I agree with the poster above. The conclusion is about those herbs that have been proven safe to consume.
 
dean.won
Thanks Received: 4
Forum Guests
 
Posts: 46
Joined: January 25th, 2013
 
 
 

Re: Q5 - Doctor: While a few alternative

by dean.won Mon May 20, 2013 2:28 am

does A weaken because if doctors are given free range to prescribe less effective(herbal) medicine, then ppl would neglect better more effective treatment?
 
joseph.carroll.555
Thanks Received: 2
Vinny Gambini
Vinny Gambini
 
Posts: 22
Joined: March 12th, 2013
 
 
 

Re: Q5 - Doctor: While a few alternative

by joseph.carroll.555 Mon May 20, 2013 6:10 pm

dean.won Wrote:does A weaken because if doctors are given free range to prescribe less effective(herbal) medicine, then ppl would neglect better more effective treatment?


Yes. And that, in turn, would weaken the conclusion (ever so slightly) which states that doctors should ALWAYS be allowed to prescribe herbals.
 
austindyoung
Thanks Received: 22
Elle Woods
Elle Woods
 
Posts: 75
Joined: July 05th, 2012
 
 
 

Re: Q5 - Doctor: While a few alternative

by austindyoung Thu May 23, 2013 2:07 pm

Uggh.... This problem.

I get what the above posters said, but I think (A) would have made much more sense if instead of "neglect" it said something like "ignore."

I felt like there was an assumption tagging along with this problem: that if a product doesn't help a patient the most (something "more effective") that it shouldn't be allowed to be prescribed. And notice how strong that is even- not prescribed- but not even *allowed* to be prescribed.

So to me, (A) only made sense if I inserted that assumption, because "neglect" does not mean "ignore." One can neglect something with full knowledge of the relative benefits. Plenty of people neglect treatments they know are more helpful for a slew of reasons.

Maybe I'm wrong. I just thought this Q had a lot of undertones.
User avatar
 
ManhattanPrepLSAT1
Thanks Received: 1909
Atticus Finch
Atticus Finch
 
Posts: 2851
Joined: October 07th, 2009
 
 
 

Re: Q5 - Doctor: While a few alternative

by ManhattanPrepLSAT1 Fri May 24, 2013 5:10 pm

Hey Austin, I guess I don't quite see your point. I see "neglect" and "ignore" as really the same thing. To me it suggests that people do not seek more effective treatments - which would be a big problem for permitting these relatively harmless, but also relatively ineffective alternative medicines.

I know it's tough to go back and forth between being very picky with language and then being very flexible. My advice would be to start with being flexible and only if you end up with more than one contending answer choice - tighten up on the subtleties of language.

Hope that helps!
 
dmsqlc1121
Thanks Received: 2
Vinny Gambini
Vinny Gambini
 
Posts: 7
Joined: January 28th, 2015
 
 
 

Re: Q5 - Doctor: While a few alternative

by dmsqlc1121 Fri Nov 13, 2015 3:42 pm

This is one of the poorly worded questions that I find so frustrating.
I chose (C) as my answer although I was skeptical due to the broadened scope.

But I was also skeptical about answer choice (A) and here is my reason for that:

Conclusion: "advocates of these herbs as remedies for serious illnesses should always be allowed to prescribe them"
Premise: "patients will not be harmed, and might be helped"

Now, I see why (A) seems like a plausible answer. It seems to weaken the very premise that the herb remedies "might be helped."

So it seems that patients' negligence to use conventional medicines that are "more effective" weakens the argument by hitting the weak spot of the premise.

However, my concern is that because the conclusion states advocates of herbal remedies "be allowed" to prescribe them, it does not interfere in arguing that they "be allowed" to do something.

Maybe I am being too strict about the wording of the conclusion. But I still think that it would have been clearer to select (A) over other answer choices if the "should always be allowed" was replaced with something like "should always select herbal remedies over conventional medicines."

I feel like I'm missing something in attacking weaken questions. Can someone clarify me on my frustration? Please. Thanks ahead of time.
User avatar
 
ManhattanPrepLSAT1
Thanks Received: 1909
Atticus Finch
Atticus Finch
 
Posts: 2851
Joined: October 07th, 2009
 
 
 

Re: Q5 - Doctor: While a few alternative

by ManhattanPrepLSAT1 Mon Nov 16, 2015 2:50 pm

Hi dmsqlc1121,

Lets compare the conclusion with your alternative.

dmsqlc1121 Wrote:Maybe I am being too strict about the wording of the conclusion. But I still think that it would have been clearer to select (A) over other answer choices if the "should always be allowed" was replaced with something like "should always select herbal remedies over conventional medicines."


Yours is much stronger. But do we need to make the conclusion stronger in order for answer choice (A) to undermine the argument? I don't think so. If answer choice (A) would undermine the weaker conclusion, then it naturally would weaken your stronger conclusion as well.

Can we think of a reason why advocates of herbal remedies should not be allowed to prescribe herbal remedies knowing that herbal remedies have little firm evidence of their effectiveness? Sure, suppose you have two medical options: the first is effective, the second is not effective. If choosing the second option would deter you from also choosing the first, then the second option should not be prescribed.

Answer choice (A) gives a clear reason why one wouldn't want to permit advocates of herbal remedies from prescribing such herbs, since they reduce the likelihood that one would consume an effective remedy.

Hope that helps!
 
obobob
Thanks Received: 1
Elle Woods
Elle Woods
 
Posts: 78
Joined: January 21st, 2018
 
 
 

Re: Q5 - Doctor: While a few alternative

by obobob Tue Mar 19, 2019 6:45 am

Hi-- just to clarify about the answer choice (C).

Since (C) does not address anything about the conclusion regarding herbs, (C) is an incorrect answer for it's irrelevant?

Can anyone confirm this?
User avatar
 
ohthatpatrick
Thanks Received: 3808
Atticus Finch
Atticus Finch
 
Posts: 4661
Joined: April 01st, 2011
 
 
 

Re: Q5 - Doctor: While a few alternative

by ohthatpatrick Thu Mar 21, 2019 3:13 pm

Any time you're doing Str or Weaken, you should be nervous about picking an answer that says "Some".

SOME = at least one

So (C) says
"There's at least one patient who might have an allergic reaction to medicine X, even though medicine X is tolerated by other patients"

Okay ..... so?

That's a pretty obvious truism, right? Even if 99% of people tolerate a certain medicine fine, there might be SOME patient that has an allergic reaction to it.

That's not going to impact this conversation much at all.

We need the strongest way of arguing that "herbal advocates shouldn't always be allowed to prescribe herbs".

Why shouldn't they always be allowed to prescribe herbs?
(A) says "it's because many patients will neglect to use a more effective medicine and will use the herb instead"

(C) says "at least one person may have an allergic reaction to a medicine (who knows if we're talking about herbal medicines or not) even though other patients don't have an allergic reaction to that medicine"


Hope this helps
 
LaraF343
Thanks Received: 0
Vinny Gambini
Vinny Gambini
 
Posts: 2
Joined: February 11th, 2019
 
 
 

Re: Q5 - Doctor: While a few alternative

by LaraF343 Mon Apr 08, 2019 2:16 pm

ohthatpatrick Wrote:Any time you're doing Str or Weaken, you should be nervous about picking an answer that says "Some".

SOME = at least one

So (C) says
"There's at least one patient who might have an allergic reaction to medicine X, even though medicine X is tolerated by other patients"

Okay ..... so?

That's a pretty obvious truism, right? Even if 99% of people tolerate a certain medicine fine, there might be SOME patient that has an allergic reaction to it.


Right, but the argument says that they should ALWAYS be allowed to prescribe them since their patients WILL NOT be harmed. If 1% of my patients are harmed, then they're still harmed.

I didn't choose A for a similar reason. Maybe the conventional medicine is 1% more effective. So what? Doctors are still allowed to prescribe the less effective option as long as it does no harm and they might be helped.
User avatar
 
ohthatpatrick
Thanks Received: 3808
Atticus Finch
Atticus Finch
 
Posts: 4661
Joined: April 01st, 2011
 
 
 

Re: Q5 - Doctor: While a few alternative

by ohthatpatrick Tue Apr 16, 2019 12:24 am

(C) would not be referring to any of the herbs that are being discussed in the conclusion, since we are told as a premise that "patients will not be harmed by the use of these products".

Whatever "certain medications" (C) is referring to must not be "these herbs", since we know that patients will not be harmed by these herbs.

If we're trying to say, "sure they won't be harmed, but they still might have an allergic reaction", I'd probably say we're splitting hairs --- isn't an allergic reaction an adverse effect / doesn't it fall under the category of harm?

If it does, then (C) can't possibly apply to our known-to-be-harmless herbs.
If it doesn't, then (C) allows us to make the objection that "at least once, we shouldn't be allowed to prescribe these herbs because a patient may have an allergic reaction to it".

Even if we read (C) as charitably as possible, you're thinking
WHAT'S A STRONGER QUANTITY:
"Many" practitioners and patients
vs.
"Some" patients

WHAT'S A STRONGER DOWNSIDE TO TAKING HERBAL REMEDIES:
failing to take a more effective medicine
vs.
maybe having an allergic reaction

On both counts (A) wins. It's not like it's a massively damaging answer. We could certainly read it in its least damaging (1% more effective) form, but its least damaging form would still be MORE weakening than (C)'s least damaging form.

Hope this helps.